Ask your own question, for FREE!
Mathematics 59 Online
OpenStudy (anonymous):

Please help! Calc! I do not understand this at all, and would greatly appreciate someone showing/explaining the steps: Find the derivative by A, evaluating the integral and differentiating the result, and B, by differentiating the integral directly. Problem: d/dt (integral) lower 0 upper t^9 ^3 (sq rt) u * du

OpenStudy (accessdenied):

\(\large \displaystyle \frac{d}{dt} \int_{0}^{t^9} \sqrt[3]{u} \; \text{d} u \) Is this what we are looking at?

OpenStudy (anonymous):

Yes :D

OpenStudy (accessdenied):

Alright. So the first method they ask you to take on is integrating it first and then taking the derivative. So let's throw the d/dt to the wind and figure out how to integrate that integral first. \( \large \displaystyle \color{#aaa}{\frac{d}{dt}} \int_{0}^{t^9} \sqrt[3]{u} \; \text{d} u \) So,do you have any guesses as to how we would approach this integral?

OpenStudy (anonymous):

find f(t), which I have as (integral) ^9 sq rt U du...but then my algebra skills are off or something cause my answer falls off

OpenStudy (anonymous):

f(x)dx= f(b)-f(a) is what they are showing

OpenStudy (accessdenied):

Ah. That is what we do with the antiderivative once we know it. But we have to find the antiderivative of the cube root of u.

OpenStudy (anonymous):

I can simplify ^9 sq rt u du to u^1/9 du but then lost as to how they get to the next step

OpenStudy (anonymous):

the example they have shows: ^5 sq rt u du -> u^1/5 du -> 5/6u^6/5 + C

OpenStudy (anonymous):

that last step is where im lost

OpenStudy (accessdenied):

Getting from the integral of u^(1/5) du to 5/6 u^(6/5) ?

OpenStudy (anonymous):

yes

OpenStudy (accessdenied):

It is the application of power rule for integrals. Do you recall this guy? \( \displaystyle \int x^n \; \text{d}x = \frac{x^{n+1}}{n + 1} + C\)

OpenStudy (anonymous):

Honestly there are so many formulas in calc2 im going nuts but vaguely yes lol

OpenStudy (accessdenied):

Oh, yea. There are a lot! Idk what learning method is easiest for you, but I heard one strategy was to use flashcards with all the formulas to get to know them well by name. Anyways, yeah, if we apply this formula for the integral of u^(1/5), we use n=1/5.

OpenStudy (anonymous):

so for the actual problem of 3 sq rt u du, that gives u^1/3 du, which gives -> u^ 1/3+1 divided by 3+1 + C?

OpenStudy (anonymous):

which gives u^4/3 / 4 + C?

OpenStudy (accessdenied):

\( \large \displaystyle \color{#aaa}{\frac{d}{dt}} \int_{0}^{t^9} u^{1/3} \; \text{d} u \) = \( \large \displaystyle \color{#aaa}{\frac{d}{dt}} \left( \frac{u^{1/3 + 1}}{1/3 + 1}+ C \right)_{0}^{t^9} \) Be careful with those numbers! Notice the 1/3 is also in the denominator entirely.

OpenStudy (anonymous):

crap i forgot the fraction, but yeah, so that gives u^4/3 divided by 4/3 +C, so doesnt that just cancel out?

OpenStudy (accessdenied):

The denominator and the power are not mixable. It would have to be a 4/3 multiplied on top of the fraction and on the bottom. Instead, notice in the previous problems how there is that 5/6 u^(6/5). 5/6 is the same as 1 / (6/5). :)

OpenStudy (accessdenied):

Or in general, \( \dfrac{a}{b} = \dfrac{1}{b/a} \)

OpenStudy (anonymous):

so to get the 4/3 out of the denominator id multiply it out so it'd be 3/4 u ^ 4/3?

OpenStudy (accessdenied):

Yep. Then you can plug in your a and b from F(b) - F(a). F(u) = 3/4 u ^(4/3) + C Technically, that C is always going to be lost in the definite integration. I just show its still there but the difference will cancel out that constant.

OpenStudy (anonymous):

so for the next step they have evaluate the f function, but they show f(t10)[the upper interval] = 5/6 t^ 12? which would translate in this problem to f(t9)= 3/4 t^ ?? where did they get 12?

OpenStudy (anonymous):

the lower limit in the example and this problem is 0 so lost as to the 12 example

OpenStudy (anonymous):

the exponent of u also just vanished

OpenStudy (accessdenied):

\( \displaystyle \large \color{#aaa}{\frac{d}{dt}} \left( F(t^9) - F(0) \right) \) and we called \(\displaystyle F(u) = \frac{3}{4} u^{4/3} + C \) So, what do you get just by plugging in u = t^9? (this is the F(t^9) part)

OpenStudy (anonymous):

3/4(t^9)^4/3 so

OpenStudy (accessdenied):

can you simplify by exponent rules?

OpenStudy (accessdenied):

for example, \( (a^{b})^c = a^{bc} \)

OpenStudy (anonymous):

wow i forget everything sorry

OpenStudy (accessdenied):

It 's okay. :) It is a lot to know, that is totally understandable.

OpenStudy (anonymous):

so 3/4 t ^12 {explains the 12 now{

OpenStudy (accessdenied):

Yep. As for the F(0), we substitute in u = 0 into F(u).

OpenStudy (anonymous):

just gives 0

OpenStudy (anonymous):

3/4(0)^4/3

OpenStudy (anonymous):

so 3/4 t^12- 0 = 3/4 t^12

OpenStudy (accessdenied):

Correct. Then all that is left over is the derivative which was saved for last. \( \displaystyle \frac{d}{dt} \left( \frac{3}{4} t^{1}2 - 0 \right) \)

OpenStudy (accessdenied):

uhh, \(t^{12} \) thought I fixed it but i guess i didn't .

OpenStudy (anonymous):

so i have to subtract out the derivative or take the derivative as the answer?

OpenStudy (accessdenied):

take the derivative for the final answer. up until this point, the derivative was just sitting outside and all our work was inside the derivative. but until now it had no added value by being written.

OpenStudy (anonymous):

ok and the basic formula for that is what again

OpenStudy (accessdenied):

The power rule for derivatives is multiply by the power and then subtract one off the exponent \( \dfrac{d}{dx} x^n = n x^{n-1} \)

OpenStudy (anonymous):

so 3/4t^ 11? no...right?

OpenStudy (accessdenied):

You subtracted from the power correctly. but the power also is multiplied on: \( \dfrac{d}{dx} x^{n} = \color{red}n x^{n-1} \)

OpenStudy (anonymous):

9t^11?

OpenStudy (accessdenied):

That looks good to me. :)

OpenStudy (anonymous):

so just 3/4*12 gives the coefficient and then subtract off exponent right

OpenStudy (accessdenied):

Yes, that is correct. For the derivative rule, the coefficient is multiplied first and then subtract from the power For the integral power rule, the power is incremented by 1 first, then divided off.

OpenStudy (anonymous):

Roger, now the second part(B) states I need to use the chain rule? Can you explain that please

OpenStudy (accessdenied):

Chain rule and also the fundamental theorem of calculus. The formal chain rule looks a bit like this: \( \dfrac{dy}{dx} = \dfrac{dy}{du} \times \dfrac{du}{dx} \) The intuitive meaning behind it is to instate a substitution to make the derivative of y with respect to u easier, while then simply multiplying on the derivative of u with respect to x.

OpenStudy (accessdenied):

Like, if you had \( y = e^{-x^2} \) This would be hard to do as-is. But if we had \(u = -x^2\), something we can use power rule with, and then \( e^{u} \) 's derivative with respect to u is just e^u again, it becomes a multiplication of two easier derivatives. e^(-x^2) * -2x

OpenStudy (accessdenied):

As for the fundamental theorem, the statement is like this: \( \displaystyle \dfrac{d}{dt} \int_{a}^{t} f(x) \ dx = f(t) \) The noteworthy part is that the integral's bounds are a constant (a) and a single variable (t).

OpenStudy (accessdenied):

does this all seem familiar so far?

OpenStudy (anonymous):

familiar but not grasped fully yes

OpenStudy (accessdenied):

Hm.. okay. I'll try my best to make the following explanation as easy as I can. So, our starting point is this: \( \displaystyle \large \frac{d}{dt} \int_{0} ^{t^9} \; u^{1/3} \; du \) I already converted the radical to u^(1/3) because it is the useful form. By comparison to the last example, what do you notice about the boundaries that is different from the original fundamental theorem of calc I statede?

OpenStudy (anonymous):

are you talking about the t^9?

OpenStudy (accessdenied):

Yes. The original fundamental theorem statement is a t, not t^9. This is where I said we should define some intermediary function, let's say w = t^9, that will take the place of the t^9. \( \displaystyle \large \frac{d}{dt} \int_{0}^{w} u^{1/3} \; du \) Then we are lining up to use the chain rule.

OpenStudy (accessdenied):

\(= \displaystyle \large \frac{dw}{dt} \times \frac{d}{dw} \int_{0} ^{w} u^{1/3} \; du \)

OpenStudy (accessdenied):

So now we have essentially neutralized our problem of that boundary not being t when we had d/dt. We changed the variable to end up having w up there and d/dw. The fundamental theorem now applies, you see?

OpenStudy (anonymous):

how did u get d/dt to dw/dt x d/dw exactly,

OpenStudy (accessdenied):

That is the chain rule step. The easiest way I can think of understanding why is, think of the dw's as if they were fractions. When you have the same factor in the numerator and denominator, they cancel right?

OpenStudy (accessdenied):

and in this case \( = \displaystyle \large \frac{\cancel{dw}}{dt} \times \frac{d}{\cancel{dw}} \int_{0} ^{w} u^{1/3} \; du \) This gets back to the last step.

OpenStudy (anonymous):

so the chain rule was dy/dx= dy/du x du/dx... so u just took the w from the integral that replaced t and put it into?

OpenStudy (accessdenied):

Basically, yeah. If we call that big integral thing just I, it looks like this: \( \dfrac{dI}{dt} = \dfrac{dI}{dw} \times \dfrac{dw}{dt} \) Multiplication is commutative, so don't mind the order being different.

OpenStudy (anonymous):

ok slightly understand continue lol

OpenStudy (accessdenied):

Now, that whole integral thing is now in the form \( \displaystyle \frac{d}{dt} \int_{0}^{t} f(x) dx \) This is what all that work was done for. Because, by the fundamental theorem of calculus, we can now simplify the integral in our problem.

OpenStudy (anonymous):

so basically that chain rule always just cancels out for us?

OpenStudy (accessdenied):

It is always set up so that it can make a cancellation, although we are going backwards from cancellation to apply chain rule. \( \dfrac{dy}{dx} = \dfrac{du}{du} \dfrac{dy}{dx} = \dfrac{dy}{du} \dfrac{du}{dx} \) Apologies that this is very imprudent of me switching variables every few seconds. :p

OpenStudy (anonymous):

not your fault. online calc2 was my own fault haha. so after this we are back to basic integral form so then?

OpenStudy (accessdenied):

So let me just put down what we had originally so we are not lost in all these variables: \( \displaystyle \large \dfrac{dw}{dt} \times \dfrac{d}{dw} \int_{0}^{w} u^{1/3} \; du \) The next step is applying the fundamental theorem of calculus' statement to this part: \( \displaystyle \large \color{#aaa}{\dfrac{dw}{dt} \times } \dfrac{d}{dw} \int_{0}^{w} u^{1/3} \; du \)

OpenStudy (accessdenied):

Originally I said that the statement looked like this. This time, I will use the consistent variables I have above. \( \displaystyle \frac{d}{d\color{green}{w}} \int_{0}^{\color{green}{w}} f(u) \; du = f(w) \) I highlight the w's green because it is good that they are the same.

OpenStudy (anonymous):

wait, so both ways give essentially the same answer? 9t^11

OpenStudy (accessdenied):

Yes. That's a good thing, because if we got a different answer... we had started from the same point, so we should naturally find the same end.

OpenStudy (anonymous):

ugh i hate calc lol

OpenStudy (anonymous):

thank you though,

OpenStudy (accessdenied):

glad to help. :p and best of luck! it does take a lot of work to become "good" at calculus, and a lot of memorizing all those rules and becoming effective at applying them. a lot of memorizing and practice. :)

Can't find your answer? Make a FREE account and ask your own questions, OR help others and earn volunteer hours!

Join our real-time social learning platform and learn together with your friends!
Can't find your answer? Make a FREE account and ask your own questions, OR help others and earn volunteer hours!

Join our real-time social learning platform and learn together with your friends!